www.vorkurse.de
Ein Projekt von vorhilfe.de
Die Online-Kurse der Vorhilfe

E-Learning leicht gemacht.
Hallo Gast!einloggen | registrieren ]
Startseite · Mitglieder · Teams · Forum · Wissen · Kurse · Impressum
Forenbaum
^ Forenbaum
Status Mathe-Vorkurse
  Status Organisatorisches
  Status Schule
    Status Wiederholung Algebra
    Status Einführung Analysis
    Status Einführung Analytisc
    Status VK 21: Mathematik 6.
    Status VK 37: Kurvendiskussionen
    Status VK Abivorbereitungen
  Status Universität
    Status Lerngruppe LinAlg
    Status VK 13 Analysis I FH
    Status Algebra 2006
    Status VK 22: Algebra 2007
    Status GruMiHH 06
    Status VK 58: Algebra 1
    Status VK 59: Lineare Algebra
    Status VK 60: Analysis
    Status Wahrscheinlichkeitst

Gezeigt werden alle Foren bis zur Tiefe 2

Navigation
 Startseite...
 Neuerdings beta neu
 Forum...
 vorwissen...
 vorkurse...
 Werkzeuge...
 Nachhilfevermittlung beta...
 Online-Spiele beta
 Suchen
 Verein...
 Impressum
Das Projekt
Server und Internetanbindung werden durch Spenden finanziert.
Organisiert wird das Projekt von unserem Koordinatorenteam.
Hunderte Mitglieder helfen ehrenamtlich in unseren moderierten Foren.
Anbieter der Seite ist der gemeinnützige Verein "Vorhilfe.de e.V.".
Partnerseiten
Weitere Fächer:

Open Source FunktionenplotterFunkyPlot: Kostenloser und quelloffener Funktionenplotter für Linux und andere Betriebssysteme
Forum "Integrationstheorie" - monotone Klassen
monotone Klassen < Integrationstheorie < Maß/Integrat-Theorie < Analysis < Hochschule < Mathe < Vorhilfe
Ansicht: [ geschachtelt ] | ^ Forum "Integrationstheorie"  | ^^ Alle Foren  | ^ Forenbaum  | Materialien

monotone Klassen: Frage (beantwortet)
Status: (Frage) beantwortet Status 
Datum: 11:51 Mo 17.11.2008
Autor: eva-marie230

Aufgabe
Welches der folgenden Mengensysteme  m ist eine monotone Klasse?
a)m:=J^-1=alle eindimensionalen Quader
b)m:={A [mm] \subset \IR^3:(1,2,3) \in [/mm] A}
c)m:={A [mm] \subset \IR^2 [/mm] :A ist kompakt}
d)m:={A  [mm] \subset \IR^2 [/mm]  :A [mm] \cap [/mm] (IR x {0} ist eine Borelmenge in IR x {0}}

Hallo,

Ich habe versucht die Definition von einer monotonen Klasse hierrauf anzuwenden,hat aber irgendwie nicht wirklich geklappt.Diese lautet:Ist [mm] X\not= \emptyset [/mm] eine Menge,so heißt  [mm] \emptyset \not= [/mm]  M [mm] \subset [/mm] P(X) eine monotone Klasse,falls für jede wachsende Folge [mm] (A_{j})_{j \in \IN} [/mm] in M und für jede fallende Folge [mm] (B_{j})_{j \in \IN} [/mm] in M sowohl [mm] \cup_{j \in \IN} A_j [/mm]
als auch [mm] \cap_{j \in \IN} B_j [/mm] in M sind.
Könnt ihr mir da vielleicht einen Anstoß geben?

LG

        
Bezug
monotone Klassen: Antwort
Status: (Antwort) fertig Status 
Datum: 13:47 Mo 17.11.2008
Autor: fred97


> Welches der folgenden Mengensysteme  m ist eine monotone
> Klasse?
>  a)m:=J^-1=alle eindimensionalen Quader
>  b)m:={ A [mm] \subset \IR^3:(1,2,3) \in [/mm]  A }


Zu b) Nimm mal eine wachsende Folge $ [mm] (A_{j})_{j \in \IN} [/mm] $ in M und  eine fallende Folge $ [mm] (B_{j})_{j \in \IN} [/mm] $ in M


Dann ist (1,2,3) in jedem [mm] A_j [/mm] und in jedem [mm] B_j. [/mm]

Dann liegt doch (1,2,3) in  $ [mm] \cup_{j \in \IN} A_j [/mm] $ und in  $ [mm] \cap_{j \in \IN} B_j [/mm] $,

also gehören  $ [mm] \cup_{j \in \IN} A_j [/mm] $  und $ [mm] \cap_{j \in \IN} B_j [/mm] $  zu M




Kommst Du nun klar mit a), c) und d)  ?


FRED


>  c)m:={A [mm] \subset \IR^2 [/mm] :A ist kompakt}
>  d)m:={A  [mm] \subset \IR^2 [/mm]  :A [mm] \cap [/mm] (IR x {0} ist eine Borelmenge in IR x {0}}
>  Hallo,
>  
> Ich habe versucht die Definition von einer monotonen Klasse
> hierrauf anzuwenden,hat aber irgendwie nicht wirklich
> geklappt.Diese lautet:Ist [mm]X\not= \emptyset[/mm] eine Menge,so
> heißt  [mm]\emptyset \not=[/mm]  M [mm]\subset[/mm] P(X) eine monotone
> Klasse,falls für jede wachsende Folge [mm](A_{j})_{j \in \IN}[/mm]
> in M und für jede fallende Folge [mm](B_{j})_{j \in \IN}[/mm] in M
> sowohl [mm]\cup_{j \in \IN} A_j[/mm]
>  als auch [mm]\cap_{j \in \IN} B_j[/mm]
> in M sind.
>  Könnt ihr mir da vielleicht einen Anstoß geben?
>  
> LG


Bezug
                
Bezug
monotone Klassen: Frage (beantwortet)
Status: (Frage) beantwortet Status 
Datum: 21:56 Mo 17.11.2008
Autor: eva-marie230

Hallo Fred,

Danke für deine Hilfe.
Zu b) Nimm mal eine wachsende Folge $ [mm] (A_{j})_{j \in \IN} [/mm] $ in M und  eine fallende Folge $ [mm] (B_{j})_{j \in \IN} [/mm] $ in M
-Ok, also [mm] vielleicht,A_j=j^3*x \Rightarrow A_1=(1,2,3),A_2=2^3*(1,2,3)=(8,16,24), A_3=(27,54,81). [/mm]
die fallende [mm] Folge:B_j=1/(j*x),B_1=(1,1/2,1/3)??? [/mm]


Dann ist (1,2,3) in jedem [mm] A_j [/mm] und in jedem [mm] B_j. [/mm]

Dann liegt doch (1,2,3) in  $ [mm] \cup_{j \in \IN} A_j [/mm] $ und in  $ [mm] \cap_{j \in \IN} B_j [/mm] $,

also gehören  $ [mm] \cup_{j \in \IN} A_j [/mm] $  und $ [mm] \cap_{j \in \IN} B_j [/mm] $  zu M

LG





Bezug
                        
Bezug
monotone Klassen: Antwort
Status: (Antwort) fertig Status 
Datum: 06:24 Di 18.11.2008
Autor: fred97


> Hallo Fred,
>  
> Danke für deine Hilfe.
>  Zu b) Nimm mal eine wachsende Folge [mm](A_{j})_{j \in \IN}[/mm] in
> M und  eine fallende Folge [mm](B_{j})_{j \in \IN}[/mm] in M
>  -Ok, also [mm]vielleicht,A_j=j^3*x \Rightarrow A_1=(1,2,3),A_2=2^3*(1,2,3)=(8,16,24), A_3=(27,54,81).[/mm]
>  
> die fallende [mm]Folge:B_j=1/(j*x),B_1=(1,1/2,1/3)???[/mm]



Was machst Du eigentlich ????????????????????????????

Eine Folge  [mm] (A_j) [/mm] von  Mengen heißt wacsend , wenn

[mm] A_1 \subseteq A_2 \subseteq A_3 \subseteq [/mm] .......................


FRED


>  
>
> Dann ist (1,2,3) in jedem [mm]A_j[/mm] und in jedem [mm]B_j.[/mm]
>  
> Dann liegt doch (1,2,3) in  [mm]\cup_{j \in \IN} A_j[/mm] und in  
> [mm]\cap_{j \in \IN} B_j [/mm],
>  
> also gehören  [mm]\cup_{j \in \IN} A_j[/mm]  und [mm]\cap_{j \in \IN} B_j[/mm]
>  zu M
>  
> LG
>  
>
>
>  


Bezug
Ansicht: [ geschachtelt ] | ^ Forum "Integrationstheorie"  | ^^ Alle Foren  | ^ Forenbaum  | Materialien


^ Seitenanfang ^
www.vorkurse.de
[ Startseite | Mitglieder | Teams | Forum | Wissen | Kurse | Impressum ]